Unsure about the OA.Would request your opinion on this ques.

This topic has expert replies
Junior | Next Rank: 30 Posts
Posts: 21
Joined: Fri Mar 27, 2009 4:11 am
The following is a CR ques I attempted but was confused between two answer choices,the one i picked was incorrect and after going through the OE,I was unsure about the validity of it.

Please let me know your answer and your reasoning of it.I will post the OA shortly.

Thanks!!!


A car manufacturer periodically discounts
certain car models to its dealers to coincide
with intensive advertising campaigns
focused on those cars. After analyzing the
results of this program, the manufacturer
found that sales of the discounted cars was
strong, but it also concluded that it could
reap greater profits if it did not hold
promotions in this way.

Which of the following statements, if true,
best accounts for the manufacturer’s
conclusion about profitability?

A. Some consumers worry that discounted
cars are more likely to be defective.

B. The car manufacturer had not been
effective in controlling the production
costs of the cars, and these rising costs
ate into the manufacturer’s profits.

C. Although dealers requested large
numbers of the cars at discounted prices,
they generally sold the cars at the normal
retail price, thereby keeping more of the
profit for themselves.

D. Many consumers buy large-ticket items,
such as cars, only when they are on sale.
E. The manufacturer’s intensive advertising
campaign did not sufficiently emphasize
the cars’ high levels of performance on
road tests.[spoiler][/spoiler]

Senior | Next Rank: 100 Posts
Posts: 49
Joined: Sat Mar 28, 2009 9:58 am
Thanked: 3 times

by svishal1123 » Fri Apr 17, 2009 10:39 pm
Alright, let me give this a try.
We need to be careful about the last part: promotions in this way. It means that the manufacturer is doubtful about the policy of REDUCING THE PRICE TO DEALERS and ADVERTISEMENT AT THE SAME TIME

I think the answer is C

A wrong because discount to dealer does not mean discount to customer

B wrong because it is out of scope. We are only dealing with the manufacturers marketing strategy. Not completely wrong though. Its relatively wrong. Lets analyze other options.

D wrong reason same as that of A

E wrong We are not told in the question anything about the content of advertisement, also, the question statement has to do something with the price and timing of advertisement not with content. The manufacturer can change the advertisement and still continue with the strategy

C correct because in the question we are told that the sales are strong. Also, if the dealers are buying the cars in bulk, it means that the turnover in general is good and there is no reason why the discount should be given. The manufacturer would rather keep the profit itself. Also the high turnover is not a result of low price since the consumer is buying the car at normal price.

OA please?

Junior | Next Rank: 30 Posts
Posts: 21
Joined: Fri Mar 27, 2009 4:11 am

by typhoonguywlblwu » Sat Apr 18, 2009 3:54 am
As I infer, the question stem states that the increase in sales was due to periodical discounts coinciding with intensive advertising campaigns.

The conclusion as stated as per the ques is 'The car dealer thinks it could
reap greater profits if it did not hold promotions in this way'

Now,the conclusion assumes that it is the promotions = advertising campaigns which are ineffective and not the discounts.Hence,in IMO we need to look for an assumption which strengthens the conclusion.


Now 'C' gives a plausible reason which strengthens the conclusion but it is directed towards the discounts rather than the advertising campaign.Hence ,i feel it is misleading.

By process of elimination,the only other statement which has any substance in it is statement A,which if put into context can imply that if the discounted cars are advertised intensively,it can lead to a wrong perception about the company and its products and hence can negatively affect the sales.

I am not entirely convinced by A,which seems to be the best fit option.However,if the question would have been worded differently by not having any mention of advertising campaigns,the answer,by a margin would be C.

Not sure if I am hallucinating or whether this ques is actually losely worded.I would appreciate your expert comments on it.

For guys who attempted this ques and just want to verify the answer OA is C indeed.

Junior | Next Rank: 30 Posts
Posts: 18
Joined: Tue Oct 28, 2008 5:43 am
Location: India

by bzg2hg » Sat Apr 18, 2009 5:34 am
C. Although dealers requested large
numbers of the cars at discounted prices,
they generally sold the cars at the normal
retail price, thereby keeping more of the
profit for themselves.


Let me try to explain the logic behind C .

Premises says that that the customers purchased lot of cars . C is saying that the dealer sold it at normal price(without discount) . If u see both these you can conclude that customers are ready to pay normal price and purchase the car. So if the manufacturer had sold it in normal price it wud have got high profit .

Hope it helps !!! B-)

Master | Next Rank: 500 Posts
Posts: 300
Joined: Sat Aug 22, 2015 10:33 am

by kris77 » Fri May 13, 2016 11:30 pm
I feel the answer will be C.